Étude de la convergence d'une série moche

La terme général de la série est $u_n = \sqrt[n+1]{n} - \sqrt[n]{n+1}$ (probablement pour $n \geq 0$, le domaine ne pose pas de problème de toute façon).
J'ai trouvé qu'elle convergeait, mais le DL était massif et confus donc je ne suis pas serein et impossible de trouver une correction. Si quelqu'un a le courage de passer derrière (ou besoin de s'entraîner pour la rentrée ;-)), ça me rassurerait.
Si tu ne sais pas par où commencer, en passant les racines en forme exponentielle et en bourrinant on peut avancer sans avoir besoin de réfléchir. Et si comme moi tu es assez fou/folle pour chercher un équivalent, j'ai trouvé $\frac{\ln(n)}{n^2}+\frac{1}{n^2}$ (avec un $O(\frac{\ln^3(n)}{n^3})$ en terme correctif).

Réponses

  • Merci @noobey, je n'ai pas encore l'habitude d'utiliser ces outils pour m'aider à travailler en autonomie (prépa oblige...).

    Aux erreurs de signe près on dirait que je m'en suis bien sorti B-)-125136
  • Déjà c'est pour $ n\ge 1$ car $\displaystyle \sqrt[0]{.\vphantom{a}}$, euh ...
    Ensuite une fois que tu as trouvé l'équivalent $-\dfrac{\ln n}{n^2}$, pas la peine d'ajouter $1$ ni aucun $O(...)$, tu peux conclure.
  • Ah oui erratum pour le domaine, j'ai confondu puissance n-ième et racine n-ième....
    Par contre je savais que ça permettait de conclure, même $O(\frac{ln(n)}{n^2})$ aurait suffi, j'ai juste arbitrairement décidé que je voulais un équivalent à deux termes :-D
  • Bon esprit !
  • Voici comment je verrais les choses, moi je préfère les $o(...)$.
    Soit $u_{n}=\sqrt[n+1]{n}-\sqrt[n]{n+1}$, alors :
    $u_n=e^{\frac{1}{n+1}\ln n}-e^{\frac{1}{n}\ln
    (n+1)}=e^{\frac{1}{n}\ln (n+1)}(e^{\frac{1}{n+1}\ln n-\frac{1}{n}\ln(n+1)}-1)$$\sim e^{v_{n}}-1$,
    avec $v_{n}=\frac{1}{n+1}\ln n-\frac{1}{n}\ln (n+1)$.

    Or : $\frac{1}{n+1}\ln n=\frac{1}{n(1+\frac{1}{n})}\ln n=\frac{1}{n}(1-\frac{1%
    }{n}+o(\frac{1}{n}))\ln n=\frac{\ln n}{n}-\frac{\ln n}{n^{2}}+o(\frac{\ln n}{%
    n^{2}})$
    et : $\frac{1}{n}\ln (n+1)=\frac{1}{n}(\ln n+\ln (1+\frac{1}{n}))=\frac{\ln n}{%
    n}+\frac{1}{n^{2}}+o(\frac{1}{n^{2}})$.

    D’où : $v_{n}=\frac{1}{n+1}\ln n-\frac{1}{n}\ln (n+1)=-\frac{\ln n}{n^{2}}+o(\frac{\ln n}{n^{2}})\sim -\frac{\ln n}{n^{2}}$,
    et enfin : $u_{n}\sim e^{v_{n}}-1\sim v_{n}\sim -\frac{\ln n}{n^{2}}$.
    Au fond, ce n'est pas si moche...
    Bonne journée.
    Fr. Ch.
  • Et il ne faut pas parler d'un « équivalent à deux termes », mais d'un développement limité (ou asymptotique) à deux termes. En effet, on a, quand $x \rightarrow 0$, entre autres : $ \ln(1+x) \sim x+ \frac {x^{1945}}{2021}$.
  • On peut simplifier un tout petit peu la démonstration de Chaurien en écrivant :

    $v_{n}=\dfrac{1}{n+1}\ln n-\dfrac{1}{n}\ln n-\dfrac{1}{n}\ln (1+1/n)=-\dfrac{\ln n}{n(n+1)}-\dfrac{1}{n}\ln (1+1/n)\sim-\dfrac{\ln n}{n^2}$

    puisque $\dfrac{1}{n}\ln (1+1/n)\sim \dfrac{1}{n^2}$ est négligeable devant $\dfrac{\ln n}{n^2}$.

    Ici les équivalents suffisent, pas besoin de développement limité.
  • Merci beaucoup @Chaurien et @jandri pour vos corrections, elles sont très instructives B-)-
    En fait mes calculs sont justes (au signe près, j'avais seulement donné l'énoncé à l'envers), en revanche ils ne sont vraiment pas du tout efficaces. Il faut que je retrouve 1 un peu de bon sens (je n'avais pas factorisé la somme de deux exponentielles...) et 2 un peu de méthode (j'avais monté l'ordre de mes DL à l'aveugle pour arriver au résultat que je cherchais).
    Effectivement, avec un peu d'ordre pour "canaliser" les calculs, ce n'est vraiment pas si moche :-D
Connectez-vous ou Inscrivez-vous pour répondre.